Giả sử là các biến ngẫu nhiên. Khi nào thì trình tự dự kiến ​​sẽ giảm lần đầu tiên?


10

Theo đề nghị trong tiêu đề. Giả sử là các biến ngẫu nhiên iid liên tục với pdf . Hãy xem xét sự kiện , , do đó là khi chuỗi giảm lần đầu tiên. Vậy thì giá trị của gì?X1,X2,Giáo dục,XnfX1X2Giáo dụcXN-1>XNN2NE[N]

Tôi đã cố gắng đánh giá trước. Tôi có Tương tự, tôi có . Khi tôi trở nên lớn, việc tính toán trở nên phức tạp hơn và tôi không thể tìm thấy mô hình. Bất cứ ai có thể đề nghị làm thế nào tôi nên tiến hành?P[N= =Tôi] P[N=4]=1

P[N= =2]= =-f(x)F(x)dx= =F(x)22|-= =12P[N= =3]= =-f(x)xf(y)F(y)dydx= =-f(x)1-F(x)22dx= =F(x)-F(x)3/32|-= =13
tôiP[N= =4]= =1số 8Tôi

Đây có phải là một câu hỏi từ một khóa học hoặc sách giáo khoa? Nếu vậy, xin vui lòng thêm [self-study]thẻ và đọc wiki của nó .
Cá bạc

1
Một gợi ý. Hãy xem xét các cấp bậc, nên được hoán vị ngẫu nhiên. Cósắp xếp các cấp bậc . Chỉ có một hoán vị trong đó đều tăng. Đối với có quan sát không phải là mức tối đa, sau đó chúng ta có thể lấy ra và đặt ở cuối để tạo ra một chuỗi tăng dần cho đến vị trí áp chót, sau đó giảm xuống. Do đó xác suất của điều này là trong số ...? Điều đó sẽ loại bạn ra với , và mà bạn đã tìm thấy và cung cấp cho bạn một công thức đơn giản để khái quát nó. Tổng số khá dễ dàng. 1 , 2 , ... , n X i n 2 n - 1 n - 1 1 / 2 1 / 3n!1,2,Giáo dục,nXTôin2n-1n-11/21/31/số 8
Cá bạc

(Và nếu bạn không thể đoán kết quả của chuỗi bạn sẽ tổng hợp để tìm giá trị trung bình, có lẽ bạn nên chạy mô phỏng của nó. Bạn sẽ nhận ra vài chữ số thập phân đầu tiên.)
Silverfish

Đó là một vấn đề từ kỳ thi tôi đã làm hôm nay. Cảm ơn bạn đã gợi ý, bây giờ tôi đã tìm ra cách giải quyết nó.
Hao The Cabbage

2
stats.stackexchange.com/questions/51429/ trên cơ bản là một bản sao. Mặc dù nó chỉ liên quan đến một phân phối thống nhất, nhưng nó gần như không đáng kể để hiển thị hai câu hỏi tương đương nhau. (Một cách: áp dụng biến đổi tích phân xác suất cho .)XTôi
whuber

Câu trả lời:


9

Nếu là một chuỗi các biến ngẫu nhiên có thể trao đổi và thì if và ony nếu . Do đó, bằng đối xứng. Do đó, . N = phút{XTôi}Tôi1N

N= =tối thiểu{n:Xn-1>Xn},
X 1X 2X n - 1 Pr ( N n ) = Pr ( X 1X 2X n - 1 ) = 1NnX1X2Xn-1E [ N ] = Σ n = 1 Pr ( N n ) = e 2,71828 ...
Pr(Nn)= =Pr(X1X2Xn-1)= =1(n-1)!,(*)
E[N]= =Σn= =1Pr(Nn)= =e2.71828Giáo dục

Người PS hỏi về bằng chứng của . Vì chuỗi có thể trao đổi, nên phải có bất kỳ hoán vị nào , chúng ta có Vì chúng ta cóhoán vị có thể, kết quả sau.π : { 1 , Hầm , n - 1 } { 1 , Lọ , n - 1 ) ) . ( n - 1 ) !(*)Pr ( X 1X 2X n - 1 ) = Pr ( X π ( 1 )X π ( 2 )X π ( n -π:{1,Giáo dục,n-1}{1,Giáo dục,n-1}

Pr(X1X2Xn-1)= =Pr(Xπ(1)Xπ(2)Xπ(n-1)).
(n-1)!
# Monte Carlo
N <- 10^6
dec <- numeric(N)
for (i in 1:N) {
    j <- 1
    x <- rnorm(1, 0, 1) # plug your favorite distribution here!
    repeat {
        j <- j + 1
        y <- rnorm(1, 0, 1)
        if (y < x) {
            dec[i] <- j
            break
        }
        x <- y
    }
}
cat(mean(dec), "\n")
# A good example of how to program C in R!

2
Tôi thích điều này - đó là một lời nhắc chúng ta thường không cần tìm cá nhân để tìm giá trị trung bình của Y và thay vào đó có thể hữu ích hơn khi đi thẳng vào . Pr ( Y y )Pr(Y= =y)Pr(Yy)
Cá bạc

+1 - nhưng điều này không thực sự trả lời câu hỏi, trong đó giả sử số lượng hữu hạn . Tuy nhiên, kỹ thuật áp dụng cho trường hợp hữu hạn một cách rõ ràng. XTôi
whuber

1
Một chút bối rối, phải không? OP đề cập đến một "trình tự". Nhưng bạn nói đúng. Nhân tiện, nó có trực quan với bạn rằng kết quả phải là "phổ quát" (theo đúng nghĩa của nó), theo nghĩa là nó không phụ thuộc vào sự phân phối của (phân phối giống hệt) ? XTôi
Zen

1
Thật ra, sự độc lập là không cần thiết. Trao đổi là đủ. Kết quả là mạnh mẽ hơn. Tôi sẽ thêm điều này vào câu trả lời của tôi.
Thiền

3
Thật trực quan khi nó phổ biến cho các biến liên tục . Một cách để làm rõ điều này là nhận ra rằng sự kiện vẫn không thay đổi khi áp dụng biến đổi tích phân xác suất, điều này làm giảm nó trong trường hợp các biến có phân phối thống nhất chung.
whuber

8

Theo đề xuất của Silverfish, tôi sẽ đăng giải pháp bên dưới. Và

P[N= =Tôi]= =P[X1X2Giáo dụcXTôi-1>XTôi]= =P[X1X2Giáo dụcXTôi-1]-P[X1X2Giáo dụcXTôi-1XTôi]= =1(Tôi-1)!-1Tôi!
P[NTôi]= =1-P[N<Tôi]= =1-(1-12!+12!-13!++1(Tôi-2)!-1(Tôi-1)!)= =1(Tôi-1)!

Do đó .E[N]= =ΣTôi= =1P[NTôi]= =ΣTôi= =11(Tôi-1)!= =e


7

Một đối số khác: chỉ có một thứ tự đang tăng lên, ngoài sốhoán vị có thể có của . Chúng tôi quan tâm đến các thứ tự tăng cho đến vị trí áp chót, sau đó giảm xuống: điều này đòi hỏi tối đa phải ở vị trí và một trong số khác ở vị trí cuối cùng. Vì có cách để chọn ra một trong những thuật ngữ đầu tiên trong chuỗi đã đặt hàng của chúng tôi và di chuyển nó đến vị trí cuối cùng, nên xác suất là: n ! X 1 , LọXTôin!X1,Giáo dục,Xnn-1n-1XTôin-1n-1

Pr(N= =n)= =n-1n!

Lưu ý , và vì vậy điều này phù hợp với kết quả tìm thấy khi tích hợp.Pr(N= =2)= =2-12!= =12Pr(N= =3)= =3-13!= =13Pr(N= =4)= =4-14!= =1số 8

Để tìm giá trị mong đợi của chúng ta có thể sử dụng:N

E(N)= =Σn= =2nPr(N= =n)= =Σn= =2n(n-1)n!= =Σn= =21(n-2)!= =Σk= =01k!= =e

(Để làm cho tổng kết rõ ràng hơn, tôi đã sử dụng ; đối với những độc giả không quen thuộc với tổng này, hãy lấy chuỗi Taylor và thay thế )k= =n-2 ex= =Σk= =0xkk!x= =1

Chúng ta có thể kiểm tra kết quả bằng mô phỏng, đây là một số mã trong R:

firstDecrease <- function(x) {
    counter <- 2
    a <- runif(1)
    b <- runif(1)
    while(a < b){
        counter <- counter + 1
        a <- b
        b <- runif(1)
    }
    return(counter)
}

mean(mapply(firstDecrease, 1:1e7))

Điều này trở lại 2.718347, đủ gần 2.71828để đáp ứng cho tôi.


-1

EDIT: Câu trả lời của tôi là không chính xác. Tôi sẽ để nó như một ví dụ về việc một câu hỏi có vẻ đơn giản như thế này dễ hiểu như thế nào.

Tôi không nghĩ toán học của bạn đúng với trường hợp . Chúng ta có thể kiểm tra điều này thông qua một mô phỏng đơn giản:P[N= =4]

n=50000
flag <- rep(NA, n)
order <- 3
for (i in 1:n) {
  x<-rnorm(100)
  flag[i] <- all(x[order] < x[1:(order-1)])==T
}
sum(flag)/n

Cung cấp cho chúng tôi:

> sum(flag)/n
[1] 0.33326

Thay đổi orderthuật ngữ thành 4 giúp chúng tôi:

> sum(flag)/n
[1] 0.25208

Và 5:

> sum(flag)/n
[1] 0.2023

Vì vậy, nếu chúng tôi tin tưởng vào kết quả mô phỏng của mình, có vẻ như mô hình là . Nhưng điều này cũng có ý nghĩa, vì điều bạn thực sự hỏi là xác suất mà bất kỳ quan sát nào được đưa ra trong một tập hợp con của tất cả các quan sát của bạn là quan sát tối thiểu (nếu chúng ta giả sử iid thì chúng ta đang giả định khả năng trao đổi và vì vậy thứ tự là tùy tiện ). Một trong số chúng phải là mức tối thiểu, và thực sự câu hỏi đặt ra là xác suất mà bất kỳ quan sát nào được chọn ngẫu nhiên là tối thiểu. Đây chỉ là một quá trình nhị thức đơn giản.P[N= =X]= =1x


1
Bạn đã hiểu sai câu hỏi một chút, nếu cách đọc của tôi là đúng - chúng ta cần cuối cùng là bất cứ thứ gì ngoại trừ tối đa (không nhất thiết là tối thiểu) trong khi đầu tiên của phải theo thứ tự tăng dần, vì vậy một ở vị trí là tối đa. Xnn-1XTôin-1
Cá bạc

Tôi nghĩ rằng đó là một chút nhiều hơn một sự giải thích sai. Bạn nói đúng, tôi không đúng.
Dalton Hance
Khi sử dụng trang web của chúng tôi, bạn xác nhận rằng bạn đã đọc và hiểu Chính sách cookieChính sách bảo mật của chúng tôi.
Licensed under cc by-sa 3.0 with attribution required.